2012 AIME I Problems/Problem 11

Revision as of 23:48, 6 July 2015 by Daniellionyang (talk | contribs) (Solution)

Problem 11

A frog begins at $P_0 = (0,0)$ and makes a sequence of jumps according to the following rule: from $P_n = (x_n, y_n),$ the frog jumps to $P_{n+1},$ which may be any of the points $(x_n + 7, y_n + 2),$ $(x_n + 2, y_n + 7),$ $(x_n - 5, y_n - 10),$ or $(x_n - 10, y_n - 5).$ There are $M$ points $(x, y)$ with $|x| + |y| \le 100$ that can be reached by a sequence of such jumps. Find the remainder when $M$ is divided by $1000.$

Solution

First of all, it is easy to see by induction that for any $P(x,y)$ in the frog's jump sequence, $x+y$ will be a multiple of $3$ and $x-y$ will be a multiple of $5.$ The base case $(x,y) = (0,0)$ obviously satisfies the constraints and if $x+y = 3n$ and $x-y = 5m,$ any of the four transformations will sustain this property:

\begin{align*} (x+7)+(y+2) = x+y+9 \rightarrow 3(n+3) &\text{ and } (x+7)-(y+2) = x-y+5 \rightarrow 5(m+1)\\ (x+2)+(y+7) = x+y+9 \rightarrow 3(n+3) &\text{ and } (x+2)-(y+7) = x-y-5 \rightarrow 5(m-1)\\ (x-5)+(y-10) = x+y-15 \rightarrow 3(n-5) &\text{ and } (x-5)-(y-10) = x-y+5 \rightarrow 5(m+1)\\ (x-10)+(y-5) = x+y-15 \rightarrow 3(n-5) &\text{ and } (x-10)-(y-5) = x-y-5 \rightarrow 5(m-1).\\ \end{align*} So we know that any point the frog can reach will satisfy $x+y = 3n$ and $x-y = 5m.$

$\noindent\makebox[\linewidth]{\rule{\paperwidth}{0.4pt}}$ $\textbf{Lemma:}$ Any point in the form $x+y = 3n$ and $x-y = 5m$ is reachable.


\textbf{Proof:}$Denote the total amounts of each specific transformation in the frog's jump sequence to be$a,$$ (Error compiling LaTeX. Unknown error_msg)b,$$ (Error compiling LaTeX. Unknown error_msg)c,$and$d$respectively. Then$x=7a+2b-5c-10d$and$y=2a+7b-10c-5d$and the equations$x+y = 9(a+b)-15(c+d) = 3n$and$x-y = 5(a-b)+5(c-d) = 5m$must be solvable in integers. But$3(a+b)-5(c+d) = n$implies$(c+d) \equiv n \mod 3$and thus$(a+b) = \lfloor{n/3}\rfloor + 2(c+d).$Similarly$(a-b)+(c-d) = m$implies$(a-b)$and$(c-d)$have the same parity. Now in order for an integer solution to exist, there must always be a way to ensure$(a+b)$and$(a-b)$have identical parities and also$(c+d)$and$(c-d)$have identical parities. The parity of$(a+b)$is completely dependent on$n,$so the parities of$(a-b)$and$(c-d)$must be chosen to match this value. But the parity of$(c+d)$can then be adjusted by adding or subtracting$3$until it is identical to the parity of$(c-d)$as chosen before, so we conclude that it is always possible to find an integer solution for$(a,b,c,d)$and thus any point that satisfies$x+y = 3n$and$x-y = 5m$can be reached by the frog.$\noindent\makebox[\linewidth]{\rule{\paperwidth}{0.4pt}}$To count the number of such points in the region$|x| + |y| \le 100,$we first note that any such point will lie on the intersection of one line of the form$y=x-5m$and another line of the form$y=-x+3n.$The intersection of two such lines will yield the point$(\frac{3n+5m}{2},\frac{3n-5m}{2}),$which will be integral if and only if$m$and$n$have the same parity. Now since$|x| + |y| = |x \pm y|,$we find that

<cmath> \begin{align*} |x + y| = |3n| \le 100 &\rightarrow -33 \le n \le 33\\ |x - y| = |5m| \le 100 &\rightarrow -20 \le m \le 20. \end{align*} </cmath>

So there are$ (Error compiling LaTeX. Unknown error_msg)34$possible odd values and$33$possible even values for$n,$and$20$possible odd values and$21$possible even values for$m.$Every pair of lines described above will yield a valid accessible point for all pairs of$m$and$n$with the same parity, and the number of points$M$is thus$34 \cdot 20 + 33 \cdot 21 = 1373 \rightarrow \boxed{373.}$

See also

2012 AIME I (ProblemsAnswer KeyResources)
Preceded by
Problem 10
Followed by
Problem 12
1 2 3 4 5 6 7 8 9 10 11 12 13 14 15
All AIME Problems and Solutions

The problems on this page are copyrighted by the Mathematical Association of America's American Mathematics Competitions. AMC logo.png